- Mon Jan 20, 2014 12:00 am
#47137
Complete Question Explanation
(The complete setup for this game can be found here: lsat/viewtopic.php?t=2909)
The correct answer choice is (A)
The question stem specifies that O is used in more windows than G. As discussed during the analysis of the third rule, G, and O are each used in one or two windows. Thus, in order to conform to the condition in this question, G must be used in exactly one window (with P in accordance with the first rule), and O must be used in exactly two windows.
At this point, many people are naturally inclined to assume that the G and O windows are separate, as in the following scenario:
However, this is impossible because from the third rule Y could not be used in a window. Thus, there must be an overlap and one of the windows must contain both G and O:
This inference proves sufficient to answer the question by using process of elimination. From the condition in the question stem, G is used in only one window, and that window must also contain P and O. Answer choices (B), (C), (D), and (E) each contain G, but none of those answers also contains P and O. Consequently, each of those answer choices is incorrect. Only answer choice (A) remains, and is thus correct.
Note that the second diagram above is incomplete. For example, the “third” window must contain Y and P (Y must be used in one of the windows and P must also be used when Y is used), and the second rule involving 2 R’s must also be accounted for. However, there is no need to continue to create the diagram when the initial inference allows you to eliminate each of the four incorrect answers.
(The complete setup for this game can be found here: lsat/viewtopic.php?t=2909)
The correct answer choice is (A)
The question stem specifies that O is used in more windows than G. As discussed during the analysis of the third rule, G, and O are each used in one or two windows. Thus, in order to conform to the condition in this question, G must be used in exactly one window (with P in accordance with the first rule), and O must be used in exactly two windows.
At this point, many people are naturally inclined to assume that the G and O windows are separate, as in the following scenario:
However, this is impossible because from the third rule Y could not be used in a window. Thus, there must be an overlap and one of the windows must contain both G and O:
This inference proves sufficient to answer the question by using process of elimination. From the condition in the question stem, G is used in only one window, and that window must also contain P and O. Answer choices (B), (C), (D), and (E) each contain G, but none of those answers also contains P and O. Consequently, each of those answer choices is incorrect. Only answer choice (A) remains, and is thus correct.
Note that the second diagram above is incomplete. For example, the “third” window must contain Y and P (Y must be used in one of the windows and P must also be used when Y is used), and the second rule involving 2 R’s must also be accounted for. However, there is no need to continue to create the diagram when the initial inference allows you to eliminate each of the four incorrect answers.
You do not have the required permissions to view the files attached to this post.